You are on page 1of 22

Universidad Nacional de Salta

Facultad de Ciencias Exactas


Departamento de Fsica

FSICA I
2015

GUA DE TRABAJOS
PRACTICOS
1 Parte
FSICA I

AO 2015
Andrea C. Monaldi

Universidad Nacional de Salta


Facultad de Ciencias Exactas
Departamento de Fsica

FSICA I
2015

Trabajo Prctico N 1: Errores de Medicin

1. Medidas indirectas: Muestre, usando propagacin de errores que:


a) El error absoluto de la suma algebraica es igual a la suma de los errores absolutos de los
trminos.
b) El error relativo de un producto es igual a la suma algebraica de los errores relativos de los
factores.
2. El tanque de agua de una casa que tiene 1000 litros de capacidad (medida con un error de 0.4%)
se llena con una bomba. Se mide el tiempo de llenado del tanque con un cronmetro de
apreciacin 0.5 s y se obtiene una lectura de 794.0 s. Determine el caudal Q de la bomba (en lt/s)
a partir de las mediciones de volumen y tiempo. Determina el error absoluto y porcentual de Q.
3. Se desea medir el volumen de una lmina circular de acrlico. Para ello se mide el espesor y se
obtiene: e = (4,12 0,01) mm. Se mide el dimetro de la misma lmina con tres instrumentos,
obtenindose los siguientes resultados:
Calibre A
Calibre B
Regla
El volumen de la lmina es V = D2 e/4

D = (101.85 0.05) mm
D = (101.8 0.1) mm
D = (102 1) mm

a) Cul de las mediciones del dimetro elegira para no desmejorar la medicin del espesor, es
decir, para que la calidad de ambas mediciones sea del mismo orden de magnitud? Explique
brevemente.
b) Exprese el volumen con su correspondiente error.
4. Si un haz de luz pasa de aire a vidrio, los ngulos de incidencia i,
y de refraccin r estn definidos como en la figura, y se

Aire

relacionan mediante la Ley de Snell: seni = nsenr; donde n es el


ndice de refraccin del vidrio puede calcularse fcilmente:
a) Encuentre la expresin para n y para su incerteza n.

Vidrio
n

b) Si se mide i=201 y r = 131. Exprese la medida del ndice


de refraccin del vidrio con su correspondiente error.
5.

Imagine que desea medir el rea de una placa rectangular. Una vez que ha encontrado el
dispositivo de medicin ms adecuado, hace varias mediciones del alto l y del ancho b de la
placa. Para considerar las irregularidades en los lados, realiza las medidas en diferentes
posiciones. Ud. realiza 10 mediciones de l y b y obtiene los siguientes resultados:
Valores medidos
l [cm]

24.25, 24.26, 24.22, 24.28, 24.24


24.25, 24.22, 24.26, 24.23, 24.24

b [cm]

Andrea C. Monaldi

50.36, 50.35, 50.41, 50.37, 50.36

Universidad Nacional de Salta


Facultad de Ciencias Exactas
Departamento de Fsica

FSICA I
2015

50.32, 50.39, 50.38, 50.36, 50.38

a) Calcule el mejor valor estimado para l y b, as como las desviaciones estndar y los errores
cuadrticos medios del promedio. Escriba los resultados de ambas mediciones.
b) Estime el rea de la placa. Escriba el resultado de la medida (con su correspondiente error).
6.

La siguiente tabla muestra los dimetros internos en pulgadas de una muestra de 60 rulemanes
fabricados por una compaa:

0,738

0,729

0,743

0,740

0,736

0,741

0,735

0,731

0,726

0,737

0,728

0,737

0,736

0,735

0,724

0,733

0,742

0,736

0,739

0,735

0,745

0,736

0,742

0,740

0,728

0,738

0,725

0,733

0,734

0,732

0,733

0,730

0,732

0,730

0,739

0,734

0,738

0,739

0,727

0,735

0,735

0,732

0,735

0,727

0,734

0,732

0,736

0,741

0,736

0,744

0,732

0,737

0,731

0,746

0,735

0,735

0,729

0,734

0,730

0,740

a) Construir una distribucin de frecuencias de los dimetros.


b) Realiza el anlisis estadstico correspondiente a los datos.
c) Estimar utilizando la Teora de Gauss el nmero de rulemanes que tienen dimetros i)
superiores a 0,732, ii) no superiores a 0,736 iii) entre 0,730 y 0,738. Verifique contando a
mano.
d) El dato 0,746 podra descartarse? Justifique la respuesta.
7. Slo 24 de 200 alumnos de un Centro miden menos de 150 cm. Si la estatura promedio de dichos
alumnos es de 164 cm Cul es su varianza?
8. En una ciudad se estima que la temperatura mxima en el mes de junio sigue una distribucin
Normal, con media 23C y desviacin estndar 5C. Calcular el nmero de das del mes en los
que se espera alcanzar mximas entre 21C y 27C.
9. Las calificaciones de los 500 aspirantes presentados a un examen para contratacin laboral, se
distribuye normalmente con media 6.5 y varianza 4.
a) Calcule la probabilidad de que un aspirante obtenga ms de 8 puntos.
b) Determine la proporcin de los aspirantes con calificaciones inferiores a 5 puntos.
c) Cuntos aspirantes obtuvieron calificaciones comprendidas entre 5 y 7.5 puntos?
10. Experimento: Relacin funcional entre dos variables en un Plano Inclinado. Un experimento
muy simple consiste en dejar caer un objeto a lo largo de un plano inclinado y estimar el tiempo
que tarda en hacerlo. Es claro que mientras ms inclinado est el plano, ms rpido recorrer la
misma distancia de cada, pero Cul es la relacin funcional entre las
variables tiempo (t) e inclinacin ()? Si bien en estas instancias Ud. no
cuenta con los conocimientos para describir esta relacin con
basamentos netamente fsicos, imagine que realiza la experiencia
dejando caer un objeto en un plano inclinado liso, en el que se puede
despreciar los efectos de friccin entre superficies, como se muestra en
la figura. Para el experimento en se realizaron las siguientes actividades:
Andrea C. Monaldi

Universidad Nacional de Salta


Facultad de Ciencias Exactas
Departamento de Fsica

i.

FSICA I
2015

Se coloca un riel sobre un plano que nos permite variar su inclinacin respecto a la
horizontal. Se midi la longitud del riel, y fue de 150 cm.

ii.

Se coloc el riel en posicin horizontal mediante el uso de un nivel, tomando esta


posicin como cero grados.

iii.

Se le dio una inclinacin al riel, y se coloc el carrito en el extremo superior marcado


para su recorrido.

iv.

Se tom el tiempo de recorrido del carrito y se registr, cinco veces.

v.

Se cambi la inclinacin del riel y se repitieron los pasos iii) y iv)

La tabla de datos siguiente se obtuvo al realizar el experimento; la tabla muestra los datos obtenidos.

Inclinacin

en Tiempo de recorrido en s

Tiempo

grados 0.01

0.19s

promedio

0.15

11.05

10.97

10.88

10.67

10.85

10.88

0.21

9.11

9.07

9.13

9.16

9.16

9.13

0.27

8.11

8.09

8.11

8.09

8.09

8.10

0.33

7.20

7.26

7.35

7.22

7.33

7.27

0.39

6.80

6.69

6.77

6.71

6.74

6.74

a) Realiza la grfica t vs , de "Tiempo de Recorrido vs Inclinacin del Plano". Parece haber


una relacin lineal entre ambas variables?
b) Realiza una grfica que en lugar de tener como ordenada t = tiempo de recorrido, tenga el
cuadrado del tiempo de recorrido t2; esto es t* = t2 y que en lugar de tener como abscisa =
ngulo de inclinacin del plano, tenga el recproco del seno del ngulo de inclinacin del
plano 1/sen (): * = 1/sen (). (Si realizaste bien tus cambios de variable debes poder
obtener una recta y determinar la relacin funcional entre t* y *).
c) Estime la recta de regresin lineal.
11. Las calificaciones de un grupo de estudiantes en un reporte del examen parcial de mitad de
trimestre (x) y del examen final (y) son las siguientes:
x

77

50

71

72

81

94

96

99

67

82

66

78

34

47

85

99

99

68

a) Grafique el diagrama de dispersin


b) Estime la recta de regresin lineal.
c) Estime la calificacin del examen final de un estudiante que obtiene 85 en el reporte del
examen parcial de mitad de trimestre.
d) Sugiere la grfica una asociacin lineal?

Andrea C. Monaldi

Universidad Nacional de Salta


Facultad de Ciencias Exactas
Departamento de Fsica

FSICA I
2015

Trabajo Practico N2: Cinemtica de una partcula


1. Una corredora da vueltas en una pista circular de 45

N
y(m)

metros de radio. En la figura se esquematiza su

50

trayectoria, vista desde arriba:


a)

20

Encontrar y representar en un diagrama los vectores

10

D
-40

A x(m)
10 20 30 40 50

-20
-20

B.
Si la corredora parte de A Cul es la distancia que
viaja entre A y E?.
d)

30

desplazamiento entre los puntos A y E; A y D; A y C; A y

c)

40

Dibuje los vectores posicin en los puntos A, B, C, D y


E.

b)

-40

En t = 8,4 s su posicin es (41,6 m; -17,2 m), y en t =


25,2 s su posicin es (17,2 m; -41,6 m). Cul es su velocidad media durante este intervalo de
este tiempo? Esquematice. Para esos instantes, qu direcciones y sentidos

tendrn las

velocidades instantneas? Coinciden con los de la velocidad media? Es posible conocer el


mdulo de la velocidad instantnea con la informacin provista?
e)

Cmo debe ser la trayectoria para que la velocidad media sea nula para algn intervalo
considerado? y para que alguna de sus componentes sea nula?

2. Un diseador de pginas Web crea una animacin en la que un punto en una pantalla de
computadora tiene una posicin: rr = 4.0cm + 2.5 cm t 2 i + 5 cm tj

s 2 s

a) Determina la magnitud y direccin de la velocidad media del punto entre t=0 y t=2.0s
b) Calcule la magnitud y direccin de la velocidad instantnea en t=0, t=1.0s y t=2.0s
c) Dibuje la trayectoria del punto en la pantalla desde t= 0 a t = 2.0 s, y muestre las velocidades
calculadas en el inciso b).
3. Los siguientes grficos representan los movimientos de 2 coches en la misma carretera.
a) Se desplazan ambos en el mismo sentido?
b) Alguna velocidad es negativa?
c) Cul es la ecuacin de cada movimiento?
d) Cul es la posicin de encuentro si lo hacen? Cmo cambia la distancia de separacin entre los
coches?

Andrea C. Monaldi

Universidad Nacional de Salta


Facultad de Ciencias Exactas
Departamento de Fsica

FSICA I
2015

x (m)

x (m)

120
20
10

t (h)
3
2
1
t (h)
4
3
2
1
4.
Un tren parte del reposo y acelera durante 10 s con un aceleracin constante de 1.20 m/s2. Despus
2

marcha a velocidad constante durante 30 s y comienza a frenar a razn de 2.40 m/seg hasta que se
detiene en la estacin siguiente.
a) Calcular la distancia total recorrida.
b) b) Graficar x(t) y v(t)
5. La

figura

muestra

el
x(m)

desplazamiento de un mvil
en la direccin x

como
a)

Describa el movimiento del

mvil en cada intervalo de

tiempo, indicando el signo

funcin

de

del

la

tiempo.

velocidad

aceleracin en cada caso.

10

12

14

16

18

20

22

24

t(s)

En qu intervalos el mvil realiza un movimiento retardado? En cules un movimiento


acelerado?. En qu instantes el mvil tiene velocidad cero? Esboce una grfica para la
velocidad en funcin del tiempo y para la aceleracin b) Cul es la posicin inicial del objeto? Y
la velocidad inicial? Determine el desplazamiento entre t=0s y t=22s c) Calcule la velocidad media
en los intervalos (0, 22) s, (0, 4) s y (8, 18) s. d) Calcule la velocidad en los instantes t = 2 s, t = 8
s y t = 14 s.
6. Dos trenes, uno de los cuales lleva una velocidad de 96.6 km/h y el otro una velocidad de 128
km/h, se dirigen uno hacia el otro en la misma va recta horizontal. Cuando estn a una distancia
de 3.22 km, ambos maquinistas ven simultneamente al tren que se les acerca y aplican sus
frenos. Si los frenos retardan a ambos trenes a razn de 0.915 m/s2,
diga si chocarn.
7. Imagine que est en la terraza de un edificio, a 46.0 m del suelo.
Una persona con una estatura de 1.80 m, camina junto al edificio a
una rapidez constante de 1.20 m/s. Si usted quiere dejar caer un
huevo sobre la cabeza del sujeto, dnde deber estar ste cuando
usted suelte el huevo? Suponga que el huevo est en cada libre.
8. Un proyectil es lanzado verticalmente hacia arriba con velocidad inicial v0. Dibuje prolijamente
todos los vectores que representan la velocidad v, para cada instante o posicin indicados a
Andrea C. Monaldi

Universidad Nacional de Salta


Facultad de Ciencias Exactas
Departamento de Fsica

FSICA I
2015

continuacin. (a) Posicin inicial, (b) Hmax / 2, (c) la mitad del tiempo de la altura mxima, (d) si
v0 = 50m/s, dibuje en escala de acuerdo a los valores de las velocidades.
9. Falla en el lanzamiento: Un cohete de 7500 kg despega verticalmente desde la plataforma de
lanzamiento con una aceleracin constante hacia arriba de 2.25 m/s2 y no sufre resistencia del
aire considerable. Cuando alcanza una altura de 525 m, sus motores fallan repentinamente y
ahora la nica fuerza que acta sobre l es la gravedad:
a) Cul es la altura mxima que alcanzar este cohete desde la plataforma de
lanzamiento?
b) Despus de que el motor falla, cunto tiempo pasar antes de que se estrelle contra la
plataforma de lanzamiento, y qu rapidez tendr justo antes del impacto?
0

10. Se batea una pelota de tal manera que sale con una direccin de 37 con la horizontal y desde
una altura de 80 cm con respecto a una terraza que se encuentra a 4 m del piso. Si la velocidad
que adquiere la pelota es de 50 m/s, calcule:
a. el tiempo de vuelo,
b. el tiempo de la altura mxima,
c. la altura mxima y
d. la velocidad con que llega al piso, expresada como vector.
e. Si hay una pared de 8 m de altura situada 71 m de distancia desde donde se lanz la
bola, diga si la bola pasar la pared.
11. Un estudiante que est a 4 m de una pared
vertical lanza contra ella una pelota. La pelota
sale de su mano a 2 m por encima del suelo con
una velocidad inicial de v=(10i+10j)m/s. Cuando
la pelota choca en la pared, se invierte la

10m/s

componente horizontal de su velocidad, mientras


10m/s

que permanece sin variar su componente vertical

2m

Dnde caer la pelota al suelo?


12. Una pelota desliza, con velocidad constante, por
4m

una superficie horizontal, tal como se muestra en


la figura.
A

B
3,0 m
4,0 m

2,2 m

a. Calcular la velocidad mnima necesaria para que no caiga en el pozo.

Andrea C. Monaldi

Universidad Nacional de Salta


Facultad de Ciencias Exactas
Departamento de Fsica

FSICA I
2015

b. Hallar VB.
c.

Dibujar la trayectoria, las velocidades VA y VB, la aceleracin total en A y B: aA y


aB , y las componentes normales y tangenciales de la aceleracin en A y en B.

13. Suponga que una partcula se mueve sobre un crculo de radio R, partiendo desde el punto A.
Para cada una de las grficas que se muestran a continuacin, determine la direccin y sentido
de la velocidad angular y de la aceleracin angular:
a. La partcula gira en sentido horario aumentando continuamente su rapidez.
Determine la direccin y sentido si la misma partcula girara en sentido antihorario aumentando su rapidez. Esquematice.
b. La partcula gira en sentido horario disminuyendo continuamente su velocidad
angular.
c.

Determine la direccin y sentido si la misma partcula girara en sentido antihorario tanto en que lo hiciese aumentando su rapidez, como disminuyendo su
rapidez. Esquematice.
vA
A

vA
A

vB
vD

vB
vD

vC
a)

vC
b)

14. Un satlite artificial gira alrededor de la Tierra, completando un ciclo en aproximadamente 90


minutos. Suponiendo que su rbita es circular, que el radio medio de la Tierra es 6.370 km, y que
la altura media del satlite sobre su superficie es 280 km: a) Determinar su velocidad tangencial.
b) Determinar la aceleracin radial o centrpeta
15. Una partcula se mueve en el sentido de la agujas del reloj sobre una circunferencia de radio 1m
con su centro en (x, y) = (1m, 0). La partcula parte del reposo en el origen en el instante t = 0. Su
rapidez crece con aceleracin constante de 2m/s2. a) Qu tiempo tardar la partcula en recorrer la mitad de la circunferencia? b) Cul es el mdulo de su velocidad en ese momento? c)
Cul es la direccin de su velocidad entonces? d) Cul es su aceleracin radial y su
aceleracin tangencial en ese instante? e) Cules son el mdulo y la direccin de la aceleracin
total cuando ha recorrido la mitad de la circunferencia?
16. Un ventilador elctrico se apaga, y su velocidad angular disminuye uniformemente de 500 rpm a
200 rpm en 4s. a) Calcular la aceleracin angular en rev/s2 y el nmero de revoluciones que el

Andrea C. Monaldi

Universidad Nacional de Salta


Facultad de Ciencias Exactas
Departamento de Fsica

FSICA I
2015

motor gir en el intervalo de 4s. b) Cuntos segundos ms tardar el motor en parar si la


aceleracin angular se mantiene constante en el valor calculado en el inciso a)?
17. Dos trenes A y B se desplazan en rieles paralelos con velocidades de 70 Km/h y 90 Km/h
respectivamente. Calcule la velocidad relativa de B con respecto al tren A, en las siguientes
situaciones:
a. Si se mueven en el mismo sentido.
b. Si se mueven en sentidos opuestos.
c. Supongamos que ambos trenes mantienen las velocidades pero que el tren B se desplaza
hacia el Este, y el tren A lo hace en direccin 30 Noreste. Calcule la velocidad relativa de A
con respecto al tren B.
18. Se tienen dos sistemas de coordenadas ortogonales S y S. En t = 0 ambos sistemas estn
superpuestos y en reposo. El origen del sistema S se mueve con velocidad u = 2m/s a lo largo
del eje x del sistema S: a) Cules son las coordenadas (x; y) en S de un punto P(x = 4,0 m; y
= 5,0 m) en S, en los instantes t1 = 1,0 s y t2 = 2, 0 s? b) Respecto al sistema S, una partcula se
mueve con velocidad vx = - 1m/s , con qu velocidad se mueve con respecto al sistema S? c)
Si la velocidad de la partcula respecto a S fuera vy = 3 m/s j, qu velocidad tendra respecto a
S? Qu velocidad debera tener la partcula en S para que estuviera en reposo respecto a S?
19. Un hombre que gua a travs de una tormenta a 80 Km/h observa que las gotas de lluvia dejan
trazas en las ventanas laterales haciendo un ngulo de 80 con la vertical. Cuando se detiene el
auto, observa que la lluvia est cayendo realmente en forma vertical.
Calcule la velocidad relativa de la lluvia con respecto al auto en las siguientes situaciones:
a) Cuando el auto est detenido.
b) Cuando el auto se desplaza a 80 km/h
Ejercicios opcionales
1. Una partcula en movimiento rectilneo recorre un espacio de 7 m antes de empezar a contar
el tiempo; cuando t = 2 s posee una velocidad de 4 m/s. La ecuacin de la aceleracin est
dada por:

a =3

m 2
m
t 1 2 . Calcular:
s4
s

a) Ecuacin de la velocidad
b) Ecuacin de la posicin
c) Velocidad media de la partcula entre t = 2 s y t = 4 s

Andrea C. Monaldi

Universidad Nacional de Salta


Facultad de Ciencias Exactas
Departamento de Fsica

FSICA I
2015

2. Se lanza una piedra hacia el aire con un ngulo por encima de la horizontal, y se desprecia la
resistencia del aire. Cul de las grficas de la figura describe mejor la rapidez v de la piedra

en funcin del tiempo t mientras est en el aire?


3. Un esquiador desliza por la pendiente de
una montaa abandonndola con una
velocidad de mdulo 20 m/s: a) Calcule
53

el tiempo que tarda en llega a la zona


horizontal. b)

Calcule a que distancia


5,3 m

del borde de la pendiente llegar.


4. Un cuerpo baja deslizando por un techo

inclinado de 30, alcanzando al final del mismo una velocidad de 10 m/s y a una altura de
200m desde el suelo. A continuacin, cae siendo arrastrado por un viento en contra que
2

causa una aceleracin horizontal de 0,5 m/s . Calcule el tiempo que demora en llegar al suelo
y el alcance.465
5. Una centrifugadora pasa de estar detenida a girar a 450 r.p.m. en 15 s. Si el radio del tambor
es de 25 cm, calcular: a) El mdulo de la aceleracin angular; b) Las vueltas que da en ese
tiempo; c) El mdulo de la velocidad angular para t=10s; d) El mdulo de la aceleracin
tangencial; e) El mdulo de la aceleracin normal para t=15s
6. Un volante cuyo dimetro es de 8 pies tiene una velocidad angular que disminuye
uniformemente de 100 rpm en t=0, hasta detenerse cuando t = 4 s. Calcular la aceleracin
tangencial y normal de un punto situado sobre el borde del volante cuando t=2s. (Nota: 1 pie
= 0.3048 m)
7. Un volante de 4 pies de radio est girando con respecto a un eje horizontal mediante una
cuerda enrollada en su borde y con un peso en su extremo. Si la distancia vertical recorrida
por el peso est dado dada por la ecuacin: x=40t2 donde x se mide en pies y t en segundos,
calcular la velocidad angular y la aceleracin angular del volante en cualquier instante.

Andrea C. Monaldi

10

Universidad Nacional de Salta


Facultad de Ciencias Exactas
Departamento de Fsica

FSICA I
2015

Trabajo Prctico N 3: Dinmica de una partcula


1. Califique de verdadero o falso las siguientes aseveraciones, justificando su repuesta:
a) El peso y la masa se refieren a la misma magnitud fsica, solo que estn expresadas en
diferentes unidades.
b) Un cuerpo sobre el que no acta ninguna fuerza est necesariamente en reposo.
c) Un cuerpo sobre el que actan varias fuerzas, puede moverse con velocidad constante.
d) Toda variacin de la velocidad de un cuerpo exige la existencia de una fuerza aplicada sobre
el mismo.
e) Cuando una mariposa golpea contra el vidrio delantero de un automvil en movimiento la
fuerza que hace la mariposa sobre el vidrio tiene la misma intensidad que la que hace el vidrio
sobre la mariposa.
f)

En un cuerpo apoyado sobre un plano horizontal la fuerza peso y la fuerza que el plano hace
sobre el cuerpo son pares de accin y reaccin.

g) Cuando un colectivo frena una fuerza nos impulsa hacia adelante.


2. Un hombre de 110 kg desciende al suelo desde una altura de 12 m sujetando una cuerda que
pasa por una polea sin friccin atada a un saco de arena de 74 kg. A qu velocidad alcanza el
hombre el suelo? Dibuje el diagrama de cuerpo libre.
3. El jugador de baloncesto Darrell Griffith salt una vez 1.2 m sin carrera. (Esto significa que subi
1.2 m despus de que sus pies se separaron del piso.) Griffith pesaba 890 N. a) Qu rapidez
tena al separarse del piso? b) Si sus pies tardaron 0.300 s en separarse del piso despus de que
Griffith inici su salto, qu aceleracin media (magnitud y direccin) tuvo mientras se estaba
empujando contra el piso? c) Dibuje su diagrama de cuerpo libre.
4. Despus de que su automvil deportivo se descompone, usted comienza a empujarlo hacia el
taller mecnico ms cercano. Cuando el auto comienza a moverse, cmo es la fuerza que usted
ejerce sobre el auto en comparacin con la que ste ejerce sobre usted? Y cuando ya va
empujando al auto con rapidez constante?
5. Una velocista de alto rendimiento puede arrancar del bloque de salida con una aceleracin casi
horizontal de magnitud 15 m/s2. Qu fuerza horizontal debe aplicar una corredora de 55 kg al
bloque de salida al inicio para producir esta aceleracin? Qu cuerpo ejerce la fuerza que
impulsa a la corredora: el bloque de salida o ella misma?
6.

Dados los sistemas de las


figuras 1 y 2, se pide dibujar
los diagramas de cuerpo libre
para cada cuerpo e indicar
los pares de interaccin o
fuerzas de accin y reaccin
entre las dos masas A y B.
Calcule la tensin de la
cuerda y las fuerzas de
accin y reaccin entre las
masas A y B.

7. Una persona de 70 kg se coloca sobre una balanza que est ubicada dentro de un ascensor.
Determinar la medicin de la misma en cada uno de los siguientes casos:
a) El ascensor arranca movindose hacia arriba con aceleracin de 1 m/s2
b) El ascensor contina subiendo a una velocidad constante de 4 m/s
2
c) El ascensor frena con aceleracin de 2 m/s
d) El ascensor est detenido
e) Se corta la cuerda que sostiene el ascensor y cae libremente
8. Sobre una superficie horizontal spera est apoyado un bloque que pesa 30 N. Se aplica al
bloque una fuerza horizontal F, cuya intensidad aumenta gradualmente desde cero. Cuando el
Andrea C. Monaldi

11

Universidad Nacional de Salta


Facultad de Ciencias Exactas
Departamento de Fsica

FSICA I
2015

mdulo de F alcanza el valor mnimo de 9,0N, el bloque comienza a deslizar. Una vez iniciado el
movimiento, se comprueba que para que el bloque deslice con velocidad constante, es necesario
reducir el mdulo de F a un valor de 6,0 N.
a) Cunto vale la fuerza de roce que la superficie de apoyo ejerce sobre el bloque cuando F=0?,
y cundo la fuerza F ha crecido hasta un valor de 7,0 N?. Cunto vale la fuerza de roce
mxima que puede generarse entre el bloque y la superficie de apoyo?
b) Determine los coeficientes de roce esttico y dinmico entre el bloque y la superficie spera.
c) Con qu aceleracin se mueve el bloque cuando F=10,0 N?
9. Se tira de un cuerpo de masa m = 1,00 kg
por medio de una cuerda, de modo tal que
se aplica al cuerpo una fuerza que es
siempre paralela a la superficie de apoyo.
El mdulo de la fuerza es 8,0 N. En el
trayecto AB, la fuerza de rozamiento vale
2,0 N y en BC es igual a 1,6 N. El cuerpo
tiene en el punto A una velocidad vA =0 m/s.

F
F
A

37o
B

a) Hacer un diagrama de cuerpo libre correspondiente al cuerpo para un punto del trayecto AB y
otro para uno del trayecto BC.
b) Analizar qu tipo de movimiento tiene el cuerpo en AB y en BC, y calcular el tiempo que tarda
en llegar al punto C, si AB=1,00 m y BC= 2,00 m.
10. Dos bloques estn unidos por una cuerda
que pasa por una polea sin rozamiento y de
masa despreciable como indica la figura 4. Si
los planos son lisos:
a) Hacia dnde se mover el sistema?
Qu tipo de movimiento realiza cada
bloque?
b) Cul es la aceleracin de los bloques?
c) Cul es la tensin de la cuerda?
d) Repetir para el caso en que hay un
coeficiente de rozamiento dinmico c = 0,2 entre los bloques y las superficies.

kg

11. Un bloque cuya masa m1 es de 3 kg est colocado encima de otro bloque de masa m2 =5 kg.
Suponer que no hay friccin entre el bloque de 5 kg y la superficie sobre la cual reposa. Los
coeficientes de friccin esttico y cintico son 0,2 y 0,1 respectivamente.
a) Cul es la mxima fuerza que puede aplicarse al bloque de 5 kg de modo de deslizar
todo el sistema y mantener los bloques juntos?
b) Cul es la aceleracin del bloque de 3 kg si la fuerza
m1
es mayor que la fuerza mxima?
c) Cul es la mxima fuerza que puede aplicarse al
F
bloque de 3 kg para que el sistema deslice
m2
mantenindose ambos bloques juntos?
d) Cul es la aceleracin del bloque de 5 kg si la fuerza
es mayor que la mxima?
12. Sobre una mesa giratoria horizontal y plana colocamos una pequea moneda. Segn se observa,
la mesa giratoria da exactamente tres revoluciones en 3,3 segundos.
a. Cul es la velocidad de la moneda cuando gira sin deslizamiento a una distancia de
5,2cm del centro de la mesa giratoria?
b. Cul es la aceleracin (en magnitud y direccin) de la moneda en esa situacin?
c. Cul es la fuerza de friccin que acta sobre la moneda en las condiciones
anteriores, si la moneda tiene una masa de 1,7 g?
d. Cul es el coeficiente de friccin esttica entre la moneda y la mesa giratoria si se
observa que la moneda se desliza fuera de la mesa giratoria cuando est a ms de
12 cm del centro de la misma?

Andrea C. Monaldi

12

Universidad Nacional de Salta


Facultad de Ciencias Exactas
Departamento de Fsica

FSICA I
2015

13. Un pndulo cnico consiste en una masa puntual sujeta por un


hilo inextensible de longitud L que describe una trayectoria como
la indicada en la figura. Determine el periodo del pndulo en
funcin de la longitud y el ngulo del cono.

2
L

14. Una curva circular de carretera est proyectada para que los
vehculos circulen por ella a 64,5 km/h. Si el radio de la curva es
de 121,9 m Cul es el ngulo correcto de sobreelevacin (o
peralte) de la carretera? Realice el diagrama del cuerpo libre del
cuerpo e indique la direccin y sentido de la aceleracin normal o
centrpeta
15. El bloque de 4.00 kg de la figura est unido a una varilla vertical
con dos cordones. Cuando el sistema gira en torno al eje de la
varilla, los cordones se extienden como se indica en el diagrama, y
la tensin en el cordn superior es de 80.0 N. a) Qu tensin hay
en el cordn inferior? b) Cuntas revoluciones por minuto (rpm) da
el sistema? c) Calcule las rpm con las que el cordn inferior pierde
toda tensin. d) Explique qu sucede si el nmero de rpm es menor
que en el inciso c).

1.25 m
2 m
4 kg

16. Considere un camino hmedo peraltado donde hay un coeficiente


de friccin esttica de 0.30 y un coeficiente de friccin cintica de
0.25 entre los neumticos y el camino. El radio de la curva es R=50
m. a) Si el ngulo de peralte es =25, qu rapidez mxima
puede tener el coche antes de resbalar peralte arriba? b) Qu
rapidez mnima debe tener para no resbalar peralte abajo?
17. Sobre una plataforma inclinada que puede
girar en torno a un eje vertical (vase figura)
hay un pequeo dado situado a 20 cm del eje.
Si la plataforma gira a 30 rpm y su ngulo es
5, a) determinar el coeficiente de rozamiento
esttico mnimo para que el dado no resbale.
b) Repetir el clculo suponiendo que el dado
se encuentra en una plataforma horizontal.

1.25 m

20 cm

18. Un auto de masa m = 800kg est movindose


con velocidad constante vx = 72km/h i respecto a
un observador situado en O, dnde ro =1000 m j.
a) Puede considerarse al auto como una
partcula en este caso? b) Encuentre el impulso
angular L del auto respecto a O en cualquier
tiempo t. c) Cunto vale el momento de rotacin
resultante sobre el auto, respecto a O, para
cualquier tiempo t? d) Encuentre el impulso
angular L del auto respecto al punto O cuando r
= 1200m i.

vx

r
r

Ejercicios opcionales

a
A

19. Qu aceleracin debe tener el carro de la figura para que el


bloque A no caiga, si el coeficiente de rozamiento entre los dos
cuerpos es ?
20. Un cuerpo cae bajo la influencia de la gravedad y una fuerza de
arrastre f=-bv. a) Demostrar que la velocidad del cuerpo es:

v=(mg/b)(1-e-bt/m) =vlim(1-e-gt/vlim)

21. Un bloque de masa m1 est sujeto a una cuerda de longitud L1 fija por un extremo. La masa se
mueve en un crculo horizontal soportada por una mesa pulida. Una segunda masa m2 se une a la
Andrea C. Monaldi

13

Universidad Nacional de Salta


Facultad de Ciencias Exactas
Departamento de Fsica

FSICA I
2015

primera mediante una cuerda de longitud L2 y se mueve tambin en un crculo como indica la
figura. Determinar la tensin en cada una de las cuerdas si el perodo de movimiento es T.

L1

m1

L2
m2

22. Una pequea tuerca de masa 100 g puede deslizarse sin friccin por
un aro de 10 cm de radio, que gira alrededor de un eje vertical a razn
de 2 vueltas por segundo, como se indica en la figura 10. a) Determinar
el valor de para el cual la tuerca est en equilibrio vertical. b) Podra
la cuenta mantenerse a la misma altura que el centro del aro?

Andrea C. Monaldi

0.100 m

14

Universidad Nacional de Salta


Facultad de Ciencias Exactas
Departamento de Fsica

FSICA I
2015

Trabajo Prctico N 4: Trabajo y Energa


1. a) En el caso de una fuerza constante en la direccin del desplazamiento Cmo puede
efectuarse el doble de trabajo con una fuerza de la mitad de la magnitud?; b) Un elevador es
subido por sus cables con rapidez constante El trabajo neto realizado sobre l es positivo,
negativo o cero?; c) Realice el diagrama de cuerpo libre de un pndulo simple Existe alguna
fuerza que no efecte trabajo? Explique; d) Una fuerza neta acta sobre un objeto y lo acelera
desde el reposo hasta una rapidez v1, efectuando un trabajo W 1. En qu factor debe aumentarse
ese trabajo para lograr una rapidez final tres veces mayor, si el objeto parte del reposo?
2. a) Esteban ejerce una fuerza constante de magnitud 210 N sobre un automvil averiado, mientras
lo empuja una distancia de 18 m. Adems, un neumtico se desinfl, as que, para lograr que el
auto avance al frente, Esteban debe empujarlo con un ngulo de 30con respecto a la direccin
del movimiento. Cunto trabajo efecta Esteban? b) Esteban empuja un segundo automvil
r
averiado con una fuerza constante F = (160 N )i (40 N ) j
El desplazamiento del automvil

r
es s = (11m)i + (14m) j Cunto trabajo efecta Esteban en este caso?
3. Suponga que sube, a velocidad constante, un bulto de 10 kg a la caja de un camin situada a una
altura de 1 m. Calcula el trabajo que realizamos en cada uno de los siguientes casos: a) Levanta
el bulto verticalmente desde el suelo hasta la caja del camin. b) Empuja el bulto por una rampa
de 30 de inclinacin sobre la que no hay rozamiento c) Empuja el bulto por una rampa de 30 de
inclinacin sobre la que el coeficiente de rozamiento es 0,1.
4. Claudia pesa 60 kgf, y viaja en un ascensor desde el piso 4 hasta planta baja. Hallar el trabajo
que realiza la fuerza que hace el piso del ascensor (normal) sobre ella, en los siguientes
tramos de 4 m de longitud cada uno: a) Arranque con aceleracin constante, de 0,5 m/s; b)
Descenso con velocidad constante de 2 m/s; c) Frenado con aceleracin constante, de 0,5 m/s.
5. Un cuerpo de 2,0 kg se desplaza a lo largo del eje,
con una velocidad v0 = 3,0m/s i, cuando se
encuentra en x = 1,0 m. La superficie de apoyo es
spera, y el coeficiente de roce dinmico entre
sta y el cuerpo es d = 0,10. Sobre el cuerpo en
movimiento est aplicada la fuerza Fx que vara
con la posicin como se muestra en el grfico. a)
Haga un diagrama de cuerpo libre mientras el
cuerpo se mueve entre x = 0 y x = 4,0 m. b) Cul
es el trabajo neto cuando el cuerpo se desplaza
desde x = 1,0 m hasta x = 4, 0m? c) Cunto ha
variado la energa cintica?

Fx(N)
6,0

2,0

4,0

x(m)

6. Ud. debe escoger entre atrapar una pelota de 0.5 kg que se mueve a 4 m/s o una de 0.1 kg que se
mueve a 20 m/s. Cul es ms fcil de atrapar?
7. Dos veleros para hielo compiten en un lago horizontal sin friccin. Los veleros tienen masas m y
2m, respectivamente; pero sus velas son idnticas, as que el viento ejerce la misma fuerza
constante sobre cada velero. Los 2 veleros parten del reposo y la meta est a una distancia s.
Cul velero cruza la meta con mayor energa cintica?
8. Por el plano inclinado de la figura se tira un cuerpo
con una velocidad de 2 m/s. Si en el tramo AB la
Andrea C. Monaldi

3m

15

Universidad Nacional de Salta


Facultad de Ciencias Exactas
Departamento de Fsica

FSICA I
2015

superficie es perfectamente lisa y sabiendo que = 0,2 en el tramo BC, calcular la distancia d
para que el cuerpo llegue al punto C con una velocidad de 3 m/s.
9. Hallar el trabajo de la fuerza F=6xyi+3x2j N a lo largo del camino ABA.
Es la fuerza conservativa? El camino AB es el tramo de parbola y=2
2x +6x, El camino BA es la recta que une el punto B de coordenadas
(2,4) con el origen.
10. Un libro de 0.6 kg se desliza sobre una mesa horizontal. La fuerza de
friccin cintica que acta sobre el libro tiene una magnitud de 1.2 N. a)
Cunto trabajo realiza la friccin sobre el libro durante el
desplazamiento de 3 m a la izquierda? Ahora el libro se desliza 3 m a la
derecha, volviendo al punto inicial. Durante este segundo
desplazamiento Qu trabajo efecta la friccin sobre el libro? Qu
trabajo total efecta la friccin sobre el libro durante el viaje redondo?
Con base en su respuesta en la parte c) Dira que la fuerza de friccin
es conservativa o no conservativa?
11. Una masa m baja deslizndose por un plano inclinado liso que forma un ngulo con la
horizontal, desde una altura vertical inicial h. a) El trabajo efectuado por una fuerza es la sumatoria
del trabajo efectuado por las componentes de la fuerza. Considere las componentes de la
gravedad paralela y perpendicular al plano. Calcule el trabajo efectuado sobre la masa por cada
componente y use estos resultados para demostrar que el trabajo efectuado por la gravedad es
exactamente el mismo que efectuara si la masa cayera verticalmente por el aire desde una altura
h. b) Use el teorema trabajo-energa para demostrar que la rapidez de la masa en la base del
plano inclinado es la misma que tendra si se hubiera dejado caer desde la altura h, sea cual fuere
el ngulo del plano. c) Use los resultados del inciso b) para obtener la rapidez de una piedra que
baja deslizndose por una colina congelada sin friccin, partiendo del reposo 15 m arriba del pie
de la colina.
12. Se requiere un trabajo de 12 J para estirar un resorte 3 cm respecto a su longitud no estirada. a)
Cul es la constante de fuerza de este resorte? b) Qu fuerza se necesita para estirar 3 cm el
resorte desde su longitud sin estirar? c) Cunto trabajo debe efectuarse para comprimir ese
resorte 4 cm respecto a su longitud no estirada, y qu fuerza se necesita para estirarlo esta
distancia?
13. Si Ud. est en un vagn en lo alto de una montaa rusa a 70m de altura (posicin A de la figura) y
comienza a caer: a) Explica las condiciones que deben cumplirse para aplicar el principio de
conservacin de la energa mecnica al vagn durante su recorrido. Indique cmo varan las
energas cintica, potencial y mecnica. b) Qu velocidad tendr cuando pase por la posicin B?
c) Podr tener la montaa rusa un pico ms alto que el de la posicin A? d) Qu trabajo ha
hecho la fuerza del motor que ha subido el vagn al comienzo hasta la posicin A, si la masa del
vagn y los ocupantes es de 600 kg.? e) Qu fuerza ha hecho el motor, si la longitud de subida
eran 100m?

Andrea C. Monaldi

16

Universidad Nacional de Salta


Facultad de Ciencias Exactas
Departamento de Fsica

FSICA I
2015

14. Una pequea bola de masa 0,6 kg se ata a una cuerda de


longitud 0,15m, conformando un pndulo. Un bloque de 0,8
kg de masa, es golpeado por el pndulo despus de haber
sido soltado desde una altura de 0.8 m. Si el bloque est
sobre una superficie horizontal con un coeficiente de roce
de 0,6 y el choque le entrega una velocidad equivalente al
80% de la velocidad adquirida por el pndulo a) Cul es la velocidad de la bola y la tensin en la
parte inferior de la oscilacin? qu distancia recorre el bloque desde que es chocado hasta que
se detiene?
15. Un pequeo bloque de masa m se desliza sin
rozamiento por una va en forma de lazo circular
como muestra la figura. El bloque parte del reposo
a una altura h por encima de la parte inferior del
lazo. a) Demuestre que para que el bloque de la
vuelta completa al rizo, debe llegar a la parte
superior del lazo con una velocidad mnima de

h
R

v = Rg , b) Qu valor mnimo debe tener h (en


trminos de R) para que el carrito d la vuelta completa al lazo?, c) Cul es la energa cintica
del bloque cuando alcanza la parte superior del lazo? d) Cul es su aceleracin en la parte
superior del lazo, suponiendo que no se sale de la va?, e) Suponga ahora que h=3.5R y R=20m,
calcule la rapidez, aceleracin radial y aceleracin tangencial de los pasajeros cuando el carrito
est en el punto C, en el extremo de un dimetro horizontal. Haga un diagrama a escala
aproximada de las componentes de la aceleracin.
16. Un paquete de 2 kg se suelta en una pendiente de 53,1 a 4 m
de un resorte largo de masa despreciable cuya constante de
fuerza es k = 120 N/m y que est sujeto a la base de la
pendiente (ver figura). Los coeficientes de friccin entre el
paquete y la pendiente son s = 0,4 k = 0,2.
a) Qu rapidez tiene el paquete justo antes de llegar al
resorte?
b) Cul es la compresin mxima del resorte?
Opcional: c) Al rebotar, cunto se acerca a su posicin inicial?

53.1

17. En la figura se ve un bloque de 10.0 kg que se suelta desde el punto A. La pista no ofrece friccin
excepto en la parte BC, de 6 m de longitud. El bloque se mueve hacia abajo por la pista, golpea
un resorte de constante elstica k=400 N/m y lo comprime 0,6 m a partir de su posicin de
equilibrio antes de quedar momentneamente en reposo. Determine el coeficiente de friccin
cintico entre la superficie BC y el bloque.

Andrea C. Monaldi

17

Universidad Nacional de Salta


Facultad de Ciencias Exactas
Departamento de Fsica

FSICA I
2015

A
3.0 m

18. Desde el extremo A de una rampa se deja caer una partcula de 250 g de masa, que desliza con
rozamiento (coeficiente =0.5) hasta llegar al punto B. En el punto B, continua su movimiento
describiendo
el
arco
de
circunferencia BCD, de 5 m de
radio (en este tramo no hay
rozamiento). Sale por el punto D,
describiendo
un
movimiento
parablico hasta que impacta en el
punto E situado sobre un plano
inclinado 30 respecto de la
horizontal. a) Calcular la velocidad
de la partcula en el punto ms
bajo C de su trayectoria circular, y
la reaccin en dicho punto. b)
Determinar el punto de impacto del
proyectil sobre el plano inclinado DE, y las componentes de la velocidad en el punto de impacto.

19. Un coche circula a la velocidad de 90 km/h durante un tramo recto de 800 m. Calcula la potencia
desarrollada por el motor del coche si la masa del coche es de 1000 kg y el coeficiente de
rozamiento entre el suelo y las ruedas es = 0,2.
20. Suponga que una partcula se mueve a lo largo de una lnea donde la energa potencial es como
la que se muestra en la Figura.
a) Los mximos en la curva se dicen puntos de equilibrio inestable, mientras que los
mnimos, puntos
de
equilibrio estable Por
qu? Explique.
b) Realice
una
grfica
aproximada de Fx(x)
c) Identifique el rango en el
que se mover la
partcula si:
i.
E>E3
ii.
E=E2
iii.
E=E1
iv.
E=E0

Ejercicios opcionales
21. Se lanza una piedra de 20 N verticalmente hacia arriba desde el suelo. Se observa que, cuando
est 15 m sobre el suelo, viaja a 25 m/s hacia arriba. Use el teorema trabajo-energa para
determinar a) su rapidez en el momento de ser lanzada y b) su altura mxima.
22. Distancia de frenado: Un automvil viaja por un camino horizontal con rapidez v0 en el instante en
que los frenos se bloquean, de modo que las llantas se deslizan en vez de rodar. a) Use el
teorema trabajo-energa cintica para calcular la distancia mnima en que puede detenerse el auto
en trminos de v0, g y el coeficiente de friccin cintica k entre los neumticos y el camino. b) En
Andrea C. Monaldi

18

Universidad Nacional de Salta


Facultad de Ciencias Exactas
Departamento de Fsica

FSICA I
2015

qu factor cambiara la distancia mnima de frenado, si i) se duplicara el coeficiente de friccin


cintica, ii) se duplicara la rapidez inicial, o iii) se duplicaran tanto el coeficiente de friccin cintica
como la rapidez inicial?
23. En un martinete, un martillo de acero con masa de 200 kg se
levanta 3 m sobre el tope de una viga en forma de I vertical, que
se est clavando en el suelo El martillo se suelta, metiendo la
viga- I otros 7.4 cm en el suelo. Los rieles verticales que guan el
martillo ejercen una fuerza de friccin constante de 60 N sobre
ste. Use el teorema trabajo-energa para determinar a) la rapidez
del martillo justo antes de golpear la viga-I y b) la fuerza media
que el martillo ejerce sobre la viga-I. Ignore los efectos del aire.
24. Imagine que pertenece a la Cuadrilla de Rescate Alpino y debe
proyectar hacia arriba una caja de suministros por una pendiente
de ngulo constante de modo que llegue a un esquiador
varado que est a una distancia vertical h sobre la base de la
pendiente. La pendiente es resbalosa pero hay cierta friccin presente con coeficiente de friccin
cintica k. Calcule la rapidez mnima que debe impartir a la caja en la base de la pendiente para
que llegue al esquiador. Exprese su respuesta en trminos de g, h, k, y .
25. Un albail ingenioso construye un dispositivo para lanzar ladrillos hasta arriba de la pared donde
est trabajando. Se coloca un ladrillo sobre un resorte vertical comprimido con fuerza constante k
=450 N/m y masa despreciable. Al soltarse el resorte, el ladrillo es empujado hacia arriba. Si un
ladrillo con masa de 1,80 kg debe alcanzar una altura mxima de 3,6 m sobre su posicin inicial,
qu distancia deber comprimirse el resorte? (El ladrillo pierde contacto con el resorte cuando
ste recupera su longitud no comprimida)
26. Una pelota pierde el 15% de su energa cintica cuando rebota en una acera de concreto. A qu
velocidad deber arrojarse verticalmente desde una altura de 12 m para que rebote a esa misma
altura? Desprecie la resistencia del aire.
27. En un accidente de trnsito, un auto golpe a un peatn y luego el conductor pis el freno para
detener el auto. Durante el juicio, el abogado del conductor aleg que este haba respetado el
lmite de velocidad de 55 km/h que indicaban los letreros. Imagine que el fiscal lo llama como
testigo experto. Su investigacin del accidente produce las mediciones siguientes: las marcas de
derrape producidas durante el tiempo en que los frenos estaban aplicados tenan una longitud de
85.3 m y el dibujo de los neumticos produce un coeficiente de friccin cintica de 0.3 con el
pavimento Dir que el conductor conduca respetando el lmite de velocidad?
28. Una cuerpo de masa m=4 kg, est sujeto por una cuerda de longitud R=2 m, gira en el plano
inclinado 30 de la figura. a) Dibuja las fuerzas sobre el cuerpo en la posicin B (ms alta) y en la
posicin A (ms baja); b) Calcula la velocidad mnima que debe llevar el cuerpo en la posicin
ms alta B, para que pueda describir una trayectoria circular; c) Calcula la velocidad con la que
debe partir de A para que llegue a B y describa la trayectoria circular; d) Calcula la tensin de la
cuerda cuando parte de A y cuando llega a B.

Andrea C. Monaldi

19

Universidad Nacional de Salta


Facultad de Ciencias Exactas
Departamento de Fsica

FSICA I
2015

Trabajo Prctico N 5: Sistema de Partculas


1. Se tienen tres objetos de 2 kg., cada uno y estn localizados del modo siguiente: a) el objeto 1
est en x = 10 cm., y = 0; el objeto 2 est en x =0, y = 10 cm. y el objeto 3 est en x =10 cm., y =
10 cm. Localizar el centro de masas.
2. Determine la posicin del CM para cada una de las siguientes configuraciones:
a) Tres partculas de igual masa ubicadas en P1 = (0 m,0 m), P2 = (2 m,1 m) y P3 = (1 m,2 m).
b) Una barra uniforme de masa M y longitud L
c) La lmina de chapa de madera de la figura 1. Su masa es de 20 kg.
d) La placa compuesta de la figura 2, con dimensiones de 22 cm x 13 cm x 2,8 cm. La mitad de la
placa est hecha de aluminio (densidad = 2,70 gr/cm3) y la otra mitad de hierro (densidad = 7,85
gr/cm3).

30

3. Se tienen tres partculas de 0,6, 0,4 y 0,8 kg de masa en


las posiciones que muestra la figura, y las velocidades
indicadas, exprese en forma vectorial la cantidad de
movimiento del sistema y la velocidad del centro de masas.
V1 = 2,5m/s, V2 = 3 kg, V3 = 1,6 m/s

y
1
2
60o

4. Dos partculas de masas 2 kg y 3 kg se mueven, con


3
relacin a un observador, con velocidades de 10 m/s, a lo
largo del eje x, y 8 m/s en un ngulo de 120 con el eje x,
respectivamente. a) Expresar cada velocidad en forma vectorial. b) Hallar la velocidad del centro
de masa. c) Expresar la velocidad de cada partcula respecto del centro de masa. d) Hallar la
cantidad de movimiento de cada partcula en el sistema centro de masa. e) Hallar la velocidad
relativa de las partculas.
5. Una masa m1 = 5,0 kg est en reposo en el origen de un sistema de coordenadas, y otra masa
m2= 5,0 kg est en reposo en el punto P(0; 4,0 m) en t = 0. A la masa m2 se le aplica una fuerza
constante F = 10,0 i N, mientras que no acta ninguna fuerza sobre m1. a) Hallar la posicin r0 del
centro de masa en t = 0. b) Hallar la aceleracin del centro de masa. c) Determinar la velocidad
del centro de masa en t = 2,0 s. d) Cunto vale la cantidad de movimiento lineal de m2 en t = 2,0
s?, y la cantidad de movimiento lineal del sistema de partculas (m1, m2) respecto al sistema de
referencia dado?, y respecto al sistema CM? e) Para t = 2,0 s, qu posicin r tiene el centro de
masas? f) Qu impulsin neta recibi el sistema en los 2,0 s? g) Calcular el impulso angular del
sistema de partculas respecto al origen del sistema de referencia en t = 0 y en t = 2,0 s.
6. Las posiciones de dos partculas A y B de masa ma = 1kg y mb = 2kg son respectivamente ra = (4
t; 3t2) y rb = (5 2t t2; 10). Determine en t = 4s la fuerza neta exterior sobre el sistema de
partculas, el momento angular y torque respecto del origen del sistema de coordenadas.
7. Demuestre que la energa cintica de un sistema de partculas puede escribirse como la suma de
dos trminos: a) la energa cintica asociada con el movimiento del centro de masa, Ec,CM = M
2
vCM , y b) la energa cintica correspondiente al movimiento de las partculas respecto al centro

Andrea C. Monaldi

20

Universidad Nacional de Salta


Facultad de Ciencias Exactas
Departamento de Fsica

FSICA I
2015

de masa Eci = mi vi2, donde vi es la velocidad de la partcula i respecto al CM. (Considere,


PARA SIMPLIFICAR, un sistema formado por slo dos partculas).
8. Un bloque de 3 kg se mueve hacia la derecha a 5 m/s y un segundo bloque de 3 kg se mueve
hacia la izquierda a 2 m/s.
a) Hallar la energa cintica total de ambos bloques en este sistema.
b) Hallar la velocidad del centro de masas del sistema formado por los dos bloques.
c) Hallar las velocidades de los dos bloques respecto del centro de masas.
d) Hallar la energa cintica del movimiento respecto al centro de masas.
9. Un baln de soccer tiene una masa de 0.40 kg e inicialmente se mueve hacia la izquierda a 20
m/s, pero luego es pateado de manera que adquiere una velocidad con magnitud de 30 m/s y
direccin de 45 hacia arriba y a la derecha. Calcule el impulso de la fuerza neta y la fuerza neta
media, suponiendo que el choque dura t = 0.010 s.
10. Un profesor de Fsica traslada el monitor de su computadora en sus brazos, a un metro de altura
por sobre el suelo. El monitor pesa 100 N. Suponga que se le escapa de las manos y va a
estrellarse contra su pie; el tiempo de colisin monitor- pie dura 0,10 s. Suponga que:
1) el monitor se detiene contra el pie del profe;
2) rebota contra l y se eleva 2,0 cm.
a) En qu caso parece que sufre ms el profe?, por qu?
b) Estime la fuerza media ejercida sobre el pie en cada caso.
11. Un hombre de masa 70 kg y un muchacho de masa 35 kg estn de pie juntos sobre una
superficie de hielo lisa en la cual es despreciable el rozamiento. Si despus que se empujen uno
al otro, el hombre se aleja con una velocidad de 0,3 m/s respecto al hielo A qu distancia
estarn entre s ambos despus de 5 s? Se conserva la energa mecnica?
12. Un muchacho de 70 kg est parado arriba de un carrito muy largo de 500 kg que se mueve con
una velocidad de 20 m/s por un camino horizontal y sin friccin. De pronto el muchacho comienza
a correr sobre el carro hacia la parte delantera del mismo con una velocidad de 10 m/s respecto
del carrito. Cul es la nueva velocidad del carrito?
13. Dos bloques deslizan el uno hacia el otro por una superficie horizontal lisa. El bloque m1 = 6,0 kg
viene de la derecha con velocidad v1 = -2,0 m/s; el otro bloque m2 = 2,0 kg, viene de la izquierda
con velocidad v2 = 8,0 m/s. Ambos efectan un choque frontal, despus del cual m2 se mueve
hacia la izquierda con velocidad v2 = -7,0 m/s.
a) Hallar el impulso lineal del sistema formado por los dos bloques antes y despus del
choque.
b) Cunto vale la velocidad del centro de masa del sistema antes del choque?, y
despus?
c) Calcular la velocidad v1, de m1 despus del choque.
d) Determinar el coeficiente de restitucin del choque. Calcular y comparar las energas
cinticas inicial y final.
14. En un choque frontal perfectamente elstico con una masa en reposo, la masa incidente
retrocede hacia atrs con una velocidad mitad de su velocidad incidente. Cul es la razn m1/m2
de sus masas?
15. Dos esferas estn suspendidas de modo tal que en su posicin de
equilibrio sus centros quedan a la misma altura. Se separa la esfera A de
la posicin inicial y se la deja caer desde una altura h contra la B, con la
que choca:
Hallar las velocidades de cada esfera despus del primer choque, las
alturas a que llegar cada una si:
a) El choque es perfectamente elstico y las masas de ambas esferas
son iguales mA=mB. Describir el comportamiento posterior del
sistema.

A
A

b) Si mA=0,1 kg y mB=0,2kg y mA es soltada desde una altura de 0,2m

Andrea C. Monaldi

21

Universidad Nacional de Salta


Facultad de Ciencias Exactas
Departamento de Fsica

FSICA I
2015

por encima de B. Hallar las alturas a las que regresarn despus del choque si la colisin es
i) inelstica con coeficiente de restitucin 0,9, ii) plstico.
16. Los dos carritos de la figura pueden moverse con rozamiento despreciable. Al carrito A, de 3 kg,
se lo tiene en reposo, a 1,8 m por encima del tramo horizontal. Al carrito B, de 2 kg, se lo
mantiene en reposo contra un resorte
de constante elstica 1800 N/m,
A
comprimindolo 40 cm a partir de su
k
estado sin deformacin. Se liberan
1.8 m
B
ambos, y corren por la pista de modo
que se encuentran en el tramo
horizontal. All se enganchan y prosiguen juntos.
a) Determinar con qu velocidad se movern despus de engancharse.
b) Si primero se dirigen hacia el resorte, hallar qu longitud mxima lo desplazarn; en caso
contrario, a qu altura mxima llegarn sobre la rampa.
c) Hallar el impulso recibido por B, y la variacin de energa cintica que experimenta:
- debido al resorte
- debido a su choque con A
17. Un bombero de 75 kg desciende por un poste mientras una fuerza de rozamiento constante de
300 N retarda su movimiento. Una plataforma horizontal de 20 kg est sostenida por un resorte
en la parte baja del poste para amortiguar la cada. El bombero se empieza a mover desde el
reposo a 4 m por encima de la plataforma. La constante del resorte es de 400 N/m. Encuentre a)
La velocidad del bombero justo antes de chocar con la plataforma y b) la mxima compresin del
resorte.
18. Una vasija en reposo explota, rompindose en tres partes. Dos partes, la primera con el doble de
masa que la segunda, se desprenden con trayectorias perpendiculares entre s, y ambas con
velocidad de 31,4 m/s. La tercera parte tiene el triple de masa que la ms liviana de las otras dos.
Halle la magnitud y la direccin de la velocidad de la tercera parte, inmediatamente despus de la
explosin.
19. Un patinador de 83 kg se desplaza por una pista helada con una velocidad de 6,2 km/h, cuando
choca con otro patinador de 55 kg que se desplazaba en direccin perpendicular y con velocidad
de 8,5 km/h.
a) Si despus de la colisin el segundo patinador se desva en un ngulo de 18 hacia la derecha
de la direccin que traa, y adquiere una velocidad de 10 km/h. A qu velocidad y en qu
direccin se desplaza el primer patinador despus de la colisin?
b) Si en el instante en que chocan los patinadores se abrazan, A qu velocidad y en qu
direccin se desplazan juntos despus de la colisin? Considere el eje x en la direccin de
desplazamiento del primer patinador, y que el segundo choca al primero desde la derecha de
ste.
20. Se produce un choque elstico entre dos discos de hockey en una mesa sin friccin. El disco A
tiene masa mA= 0,500 kg, y el B, mB=0,300 kg. El disco A tiene velocidad inicial de 4,00 m/s en la
direccin +x y velocidad final de 2,00 m/s en direccin desconocida. El disco B est en reposo.
Calcule la rapidez final vB2 del disco B y los ngulos de desviacin respecto del eje x.
21. Un cohete est en el espacio exterior, lejos de cualquier planeta, cuando enciende su motor. En
el primer segundo de encendido, el cohete expulsa de 1/120 su masa con rapidez relativa de
2400 m/s. a) Cul es la aceleracin inicial del cohete? b) Suponga que de la masa inicial m0 del
cohete es combustible, de manera que la masa final es m=m0/4, y el combustible se consume
totalmente a ritmo constante en un tiempo t = 90 s. Si el cohete parte del reposo en nuestro
sistema de coordenadas, calcule su rapidez final al cabo de ese tiempo.

Andrea C. Monaldi

22

You might also like